가측 기수

위키백과, 우리 모두의 백과사전.

집합론에서 가측 기수(可測基數, 영어: measurable cardinal)는 기본 매장으로 정의될 수 있는 기수이다. 큰 기수의 하나이다.

정의[편집]

필터와 측도[편집]

음이 아닌 확장된 실수의 집합 에 대하여, 다음을 정의하자.[1]:129, (10.10)

임의의 기수 -완비 불 대수 에 대하여, 함수 가 다음 조건들을 모두 만족시키면, 조건을 만족시킨다고 하자.

  • 이다.
  • (단조성) 라면 이다.
  • (-가법성) 임의의 에 대하여, 만약 이며 임의의 에 대하여 라면, 이다.

이 조건을 만족시키는 함수 위의, 값의 -가법 측도(영어: -valued -additive measure on )라고 하자.

이 개념은 다음 개념들을 일반화한다.

  • 측도: 시그마 대수 위의 측도 를 만족시키는 함수이다.
  • 극대 필터: 위의 극대 필터 에 대하여, 함수 , 를 정의하면, 를 만족시킨다.

가측 기수[편집]

기수 에 대하여, 다음 세 조건이 서로 동치이며, 이를 만족시키는 기수를 가측 기수라고 한다.

  • 크기가 인 집합 위에, 주 필터가 아닌 -완비 극대 필터가 존재한다.[1]:127, Definition 10.3[2]:26, §1.2
  • 멱집합 위에, 를 만족시키는 함수 가 존재하며, 또한 임의의 에 대하여 이다. (이는 위 조건과 자명하게 동치이다.)
  • 폰 노이만 전체 로부터 ZFC의 표준 추이적 모형 으로 가는 기본 매장임계점이다.

임의의 기수 에 대하여, 만약 를 만족시키는 확률 측도 가 존재하며, 또한 다음 두 조건이 추가로 성립한다고 하자.

  • 만약 한원소 집합을 0으로 대응시킨다. (즉, 임의의 에 대하여 이다.)

그렇다면 실가 가측 기수(實價可測基數, 영어: real-valued measurable cardinal)라고 한다.[1]:130, Definition 10.8[2]:24, §2.1

성질[편집]

함의 관계[편집]

선택 공리를 추가한 체르멜로-프렝켈 집합론에서, 모든 가측 기수는 도달 불가능한 기수이며 또한 약콤팩트 기수이다.[1]:Lemma 10.18 (그러나 선택 공리를 가정하지 않으면, 가측 기수가 따름기수일 수 있다.) 모든 강콤팩트 기수는 가측 기수이다.[1]:136, §10 즉, 다음과 같은 함의 관계가 존재한다.

초콤팩트 기수강콤팩트 기수 ⇒ 가측 기수 ⇒ 약콤팩트 기수말로 기수도달 불가능한 기수정칙 기수기수순서수

모든 가측 기수는 실가 가측 기수이다. 가측 기수가 아닌 임의의 실가 가측 기수는 이하이다.[1]:131, Corollary 10.10

논리적 성질[편집]

가측 기수 에 대하여, (크기가 미만인 집합들로 구성된 폰 노이만 전체의 부분 집합)는 선택 공리를 추가한 체르멜로-프렝켈 집합론(ZFC)의 모형이다. 따라서, ZFC가 무모순적이라면 ZFC에서는 가측 기수의 존재를 증명할 수 없다. 또한, 에서는 가측 기수가 존재하지 않으므로, 적어도 하나의 가측 기수가 존재한다면 ZFC + "가측 기수의 부재"는 무모순적이다.

만약 적어도 하나 이상의 비가산 가측 기수가 존재한다면, 구성 가능성 공리 은 거짓이다.[3]

울람 행렬[편집]

임의의 두 기수 가 주어졌다고 하자.

-울람 행렬은 다음 두 성질을 만족시키는 함수

이다.[1]:131, Definition 10.11; 132, (10.14)

  • 각 열의 성분들은 서로소이다. 즉, 만약 라면, 임의의 에 대하여
  • 각 행의 성분들의 합집합여집합의 크기는 이하이다. 즉, 임의의 에 대하여, 이다.

만약 가 주어지지 않았다면, 을 뜻한다.

존재[편집]

임의의 기수 에 대하여, -울람 행렬이 존재한다.

증명:

에 대하여, 전사 함수

를 고르자. 그렇다면, 행렬 를 다음과 같이 정의하자.

그렇다면, -울람 행렬을 이룬다.

  • 임의의 에 대하여, 가 되는 유일한 이다.
  • 에 대하여, 이다.

측도와의 관계[편집]

측도 원자(영어: atom)는 이지만 임의의 에 대하여 이 되는 원소 이다.

임의의 기수 가 주어졌다고 하자.

-울람 행렬이 존재한다면, 위의 임의의 -가법 확률 측도 는 항상 공집합이 아닌 원자를 갖는다. 즉,

가 존재한다.

증명:

가 울람 행렬이라고 하고, 위의 -가법 확률 측도라고 하자. 귀류법을 사용하여, 임의의 에 대하여 라고 하자.

그렇다면, 울람 행렬의 정의에 따라, 각 에 대하여,

이다. 따라서, 가 존재한다. 이제, 각 에 대하여

를 생각하자. 이므로, 가 존재한다.

이제,

개의, 양의 측도의 서로소 집합들의 족이다. 이제, 다음 부분 집합들을 정의하자.

그렇다면,

이므로, 이 가운데 가 존재한다. 따라서,

인데, 이는 확률 측도 조건과 모순이다.

특히, 위의 임의의 (-가법) 확률 측도는 원자를 갖는다. 따라서, 만약 연속체 가설이 성립한다면, 실수선 위에, 모든 집합이 가측 집합이며, 원자가 존재하지 않는 확률 공간 구조는 존재하지 않는다.[1]:133, Corollary 10.17

역사[편집]

가측 기수는 스타니스와프 울람이 1930년에 도입하였고, 가장 작은 가측 기수가 (만약 존재한다면) 도달 불가능한 기수임을 증명하였다.[4][5]

실가 가측 기수는 스테판 바나흐가 1930년에 도입하였다.[6][1]:138, §10

참고 문헌[편집]

  1. Jech, Thomas (2003). 《Set theory》. Springer Monographs in Mathematics (영어) 3판. Springer-Verlag. doi:10.1007/3-540-44761-X. ISBN 978-3-540-44085-7. ISSN 1439-7382. Zbl 1007.03002. 
  2. Kanamori, Akihiro (2003). 《The higher infinite: large cardinals in set theory from their beginnings》. Springer Monographs in Mathematics (영어) 2판. Springer-Verlag. doi:10.1007/978-3-540-88867-3. ISBN 978-3-540-88866-6. ISSN 1439-7382. Zbl 1022.03033. 
  3. Scott, Dana S. (1961). “Measurable cardinals and constructible sets”. 《Bulletin de l’Académie polonaise des sciences. Série des Sciences mathématiques, astronomiques et physiques》 (영어) 9: 521–524. ISSN 0001-4117. Zbl 0154.00702. 
  4. Ulam, Stanisław (1930). “Zur Masstheorie in der allgemeinen Mengenlehre”. 《Fundamenta Mathematicae》 (독일어) 16 (1): 140–150. ISSN 0016-2736. 
  5. Mycielski, Jan (1987). “Learning from Ulam: measurable cardinals · ergodicity · biomathematics” (PDF). 《Los Alamos Science》 (영어) 15: 107–113. 2014년 9월 9일에 원본 문서 (PDF)에서 보존된 문서. 2014년 12월 22일에 확인함. 
  6. Banach, Stefan (1930). “Über additive Massfunktionen in abstrakten Mengen” (PDF). 《Fundamenta Mathematicae》 (독일어) 15: 97–101. ISSN 0016-2736. JFM 56.0920.03. 2016년 9월 20일에 원본 문서 (PDF)에서 보존된 문서. 2016년 9월 10일에 확인함. 

외부 링크[편집]